LSAT and Law School Admissions Forum

Get expert LSAT preparation and law school admissions advice from PowerScore Test Preparation.

User avatar
 Dave Killoran
PowerScore Staff
  • PowerScore Staff
  • Posts: 5852
  • Joined: Mar 25, 2011
|
#86000
Complete Question Explanation
(The complete setup for this game can be found here: lsat/viewtopic.php?f=303&t=7341)

The correct answer choice is (B)

Our analysis of the distribution shows that I, L, K, and G cannot attend class with P. This information eliminates answer choices (A), (C), and (D) from further consideration.

Because one of N or S must attend the first class alone, answer choice (E) can be eliminated.

Thus, answer choice (B) is correct.
 SherryZ
  • Posts: 124
  • Joined: Oct 06, 2013
|
#12867
Hi there, thank you for your help and HAPPY THANKSGIVING!

Oct 1999 LSAT, Sec 3 Game #4, Q22:

Why can't P pair with N and S?? I thought P can pair with O, N, S, H

Thank you so much!!

---Sherry
User avatar
 KelseyWoods
PowerScore Staff
  • PowerScore Staff
  • Posts: 1079
  • Joined: Jun 26, 2013
|
#12873
Happy Thanksgiving Sherry!

P could pair with N or S, but not both. You always have to have either N or S in the 1st class.** So if you put both of them with P, there is no one left who can go 1st.

(**K can't go first because the 3rd rule says she's not the first student. G/H/I can't go first because the 3rd rule says that K is the first female. L can't go 1st because the 1st rule says L has to go with I. P can't go first because the 4th rule says that G has to be before P. O can't go first because the 5th rule says that G is before O.)

Hope that helps!

Best,
Kelsey

Get the most out of your LSAT Prep Plus subscription.

Analyze and track your performance with our Testing and Analytics Package.